2.2. NÚCLEO, NULIDAD, IMAGEN Y RANGO: definiciones, ejemplos y propiedades

Por Jennyfer Paulina Bennetts Castillo

(Trabajo de titulación asesorado por la Dra. Diana Avella Alaminos)

INTRODUCCIÓN

Analizaremos cuatro nuevos conceptos. Dos de ellos son conjuntos y los otros dos son las dimensiones de esos conjuntos.

Representación gráfica del núcleo y la imagen de una transformación $T$.

NÚCLEO E IMAGEN DE UNA TRANSFORMACIÓN LINEAL

Definición: Sean $V$ y $W$ $K$ – espacios vectoriales y $T\in\mathcal{L}(V,W)$.
El núcleo de $T$ es $Núc\,T=\{v\in V|T(v)=\theta_W\}$.
La imagen de $T$ es $Im\, T=\{T(v)|v\in V\}$.

  • Sean $K$ un campo y $T:K^\infty\longrightarrow K^\infty$ lineal donde $\forall (x_1,x_2,x_3,…)\in K^\infty (T(x_1,x_2,x_3,…)=(x_2,x_3,x_4,…))$.
    $Núc\,T=\{(x_1,0_K,0_K,…)\in K^\infty | x_1\in K\}$ ; $Im\,T=K^\infty$

Justificación. Para el núcleo de $T$:

\begin{align*} T(x_1,x_2,x_3,…)=(0_K,0_K,0_K,…) \Leftrightarrow \\ (x_2,x_3,x_4,…)=(0_K,0_K,0_K,…)\Leftrightarrow \\x_i=0_K \text{ para toda }i\in\{2,3,4,…\}. \end{align*}


Para la imagen de $T$:

Sea $(y_1,y_2,y_3,…)\in K^\infty$. Tenemos que $T(0_K,y_1,y_2,…)=(y_1,y_2,y_3,…)$, por lo cual $T$ es suprayectiva y su imagen es todo el codominio.

  • Sea $T:\mathbb{R}^2\longrightarrow\mathbb{R}^2$ donde $\forall (x,y)\in\mathbb{R}^2(T(x,y)=(x,0))$
    $Núc\,T=\{(0,y)\in\mathbb{R}^2|y\in\mathbb{R}\}$ ; $Im\,T=\{(x,0)\mathbb{R}^2|x\in\mathbb{R}\}$

Justificación. Para el núcleo de $T$:

$$T(x,y)=(0,0) \Leftrightarrow (x,0)=(0,0)\Leftrightarrow x=0.$$

Para la imagen de $T$:

Sea $(a,0)\in \{ (x,0)\in\mathbb{R}^2|x\in\mathbb{R}^2\}$. Dado que $T(a,0)=(a,0)$ se tiene que $(a,0)\in Im\,T$. A la inversa, si $(a,b)\in Im\, T$ se tiene que $T(x,y)=(a,b)$ para alguna $(x,y)\in \mathbb{R}^2$, por lo que $(x,0)=(a,b)$ y así $b=0$.

  • Sean $K$ un campo, $A\in\mathcal{M}_{m\times n}(K)$ y $T:K^n\longrightarrow K^m$ donde $\forall X\in K^n(T(X)=AX)$
    $Núc\,T$ es el conjunto de las soluciones del sistema homogéneo con matriz de coeficientes $A$ ; $Im\,T$ es el espacio generado por las columnas de $A$

Justificación. Para el núcleo de $T$:

$$T(X)=\theta_{m\times 1}\Leftrightarrow AX=\theta_{m\times 1} \Leftrightarrow X \text{ es solución del sistema homogéneo con matriz de coeficientes }A.$$


Para la imagen de $T$:

\begin{align*}Im\,T&=\{AX:X\in K^n\}\\&=\left\{ \begin{pmatrix} a_{11} & … & a_{1n} \\ \vdots & \ddots & \vdots \\ a_{m1} & … & a_{mn} \end{pmatrix} \begin{pmatrix} x_1\\ \vdots \\ x_n \end{pmatrix} : x_1,x_2,…,x_n\in K \right\}\\&=\left\{ \begin{pmatrix} a_{11}x_1 + … + a_{1n}x_n \\ … \\ a_{m1}x_1 + … + a_{mn}x_n \end{pmatrix} : x_1,x_2,…,x_n\in K \right\}\\&=\left\{ x_1\begin{pmatrix} a_{11}\\ \vdots \\ a_{m1} \end{pmatrix} + … + x_n\begin{pmatrix} a_{1n}\\ \vdots \\ a_{mn} \end{pmatrix} : x_1,x_2,…,x_n\in K \right\}\\&=\left\langle \begin{pmatrix} a_{11}\\ \vdots \\ a_{m1} \end{pmatrix},…,\begin{pmatrix} a_{11}\\ \vdots \\ a_{m1} \end{pmatrix} \right\rangle\end{align*}

Proposición: Sean $V,W$ $K$ – espacios vectoriales, $T\in\mathcal{L}(V,W)$. Se cumple que:

a) $Núc\,T\leqslant V$.
b) $Im\,T\leqslant W$.

Demostración: Para cada inciso es necesario demostrar dos propiedades:

a) P.D. $\theta_V\in Núc\,T$ y $\forall\lambda\in K$ $\forall u,v\in Núc\,T (\lambda u + v\in Núc\,T)$

Como $T$ es una transformación lineal tenemos que $T(\theta_V)=\theta_W$, por lo tanto, $\theta_V\in Núc\,T.$

Sean $\lambda\in K$ y $u,v\in Núc\,T$. Entonces $T(u)=\theta_W=T(v).$ Además, $T(\lambda u+v)=\lambda T(u)+T(v)$ por ser $T$ lineal. Así, $$T(\lambda u+v)=\lambda\theta_W +\theta_W=\theta_W$$
de donde $\lambda u + v\in Núc\,T.$

b) P.D. $\theta_W\in Im\,T$ y $\forall\lambda\in K$ $\forall w,z\in Im\,T (\lambda u + v\in Im\,T)$

Como $T$ es una transformación lineal tenemos que $\theta_V\in V$ cumple que $T(\theta_V)=\theta_W$, por lo tanto, $\theta_W\in Im\,T$.

Sean $\lambda\in K$ y $w,z\in Im\,T$. Entonces $\exists u,v\in V (T(u)=w\wedge T(v)=z)$. Además, $T(\lambda u+v)=\lambda T(u)+T(v)$ por ser $T$ lineal.
Así, $$T(\lambda u+v)=\lambda w+z$$
de donde $\lambda w+ z\in Im\,T.$

NULIDAD Y RANGO DE UNA TRANSFORMACIÓN LINEAL

Definición: Sea $T$ una transformación lineal con $Núc \,T$ de dimensión finita. Decimos que la dimensión de $Núc\,T$ es la nulidad de $T$.

Definición: Sea $T$ una transformación lineal con $Im \,T$ de dimensión finita. Decimos que la dimensión de $Im\,T$ es el rango de $T$.

Ejemplo

  • Sea $K=\mathbb{R}$ y sean $V=\mathcal{P}_3$ y $W=\mathcal{P}_2$ $K$ – espacios vectoriales.
    Sea $T:V\longrightarrow W$ donde $\forall p(x)\in T(p(x))=p'(x)$.
    La nulidad de $T$ es $1$ y su rango es $3$

Justificación. Los polinomios con derivada cero son únicamente las constantes. Así, $Núc(T)=\{a|a\in\mathbb{R}\}$ que tiene dimensión $1$.

Por otro lado todo polinomio de grado $2$ se puede obtener derivando un polinomio de grado $3$. Basta con integrar el polinomio de grado $2$ para encontrar cómo son los polinomios de grado $3$ que cumplen lo deseado. De modo que $W\subseteq Im(T)$ y como $Im(T)\subseteq W$ por definición, entonces $Im(T)=W$ que tiene dimensión $3$.

Por lo tanto, el núcleo y la imagen son de dimensión finita y la nulidad de $T$ es $1$ y su rango es $3.$

Tarea Moral

  1. Sean $K$ un campo, $V$ y $W$ $K$-espacios vectoriales y $T:V\longrightarrow W$ lineal. Sea $\{ w_1, w_2, …, w_k\}$ un subconjunto l.i. de $Im\,T$.
    Si $S=\{ v_1,v_2,…,v_k \}$ se selecciona de tal forma que $\forall i\in \{ 1,2,…,k\}(T(v_i)=w_i)$, demuestra que $S$ es l.i.
  2. Para la transformación lineal $T:\mathbb{R}^3\longrightarrow \mathbb{R}^2$ con $T(a_1,a_2,a_3)=(a_1 + 2a_2, 2a_3 – a_1)$ encuentra bases para $Núc(T)$ e $Im(T)$.
  3. Sean $K$ un campo y $P: \mathcal{M}_{m\times m}(K) \longrightarrow \mathcal{M}_{m\times m}(K)$ definida por $\forall A\in \mathcal{M}_{m\times m}(K) \left( P(A)=\frac{A + A^{t}}{2} \right)$. Verifica que $T$ es lineal y encuentra su núcleo e imagen.

Más adelante…

En la siguiente entrada veremos el vínculo que existe entre la dimensión del núcleo, de la imagen y del espacio vectorial que aparece como dominio de una transformación lineal. Esta relación numérica nos permite calcular cualquiera de estas dimensiones si tenemos conocimiento de las otras dos.

Entradas relacionadas

Tres versiones del Tema de la Función Inversa

Por Angélica Amellali Mercado Aguilar

Teorema de la Función Implicita (version (1))

Teorema 1. Considere la función $y=f(x)$. Sea $(x_{0},y_{0}) \in
\mathbb{R}^{2}$ un punto tal que $F(x_{0},y_{0})=0$. Suponga que la función F tiene derivadas parciales en alguna bola con centro $(x_{0},y_{0})$ y que $\displaystyle \frac{\partial F}{\partial y}(x_{0},y_{0})\neq 0$. Entonces $F(x,y)=0$ se puede resolver para $y$ en términos de $x$ y definir así una función $y=f(x)$ con dominio en una vecindad de $(x_{0},y_{0})$, tal que $y_{0}=f(x_{0})$, lo cual tiene derivadas continuas en $\mathcal{V}$ que pueden calcularse como $$y’=f'(x)=-\displaystyle \frac{\displaystyle \frac{\partial F}{\partial x}(x,y)}{\displaystyle \frac{\partial F}{\partial y}(x,y)}$, $x \in \mathcal{V}$$.

Vamos ahora a probar que f es continua en $(x_{0}-h,x_{0}+h)$ haciendo ver primero que es continua en$x_{0}$ y despues mostrando que es continua en todo $x\in (x_{0}-h,x_{0}+h)$

Demostración. Sea $0<\epsilon<k$. Si se repite el proceso para determinar la funcion f, pero ahora restringidos a un cuadrado más pequeño T, centrado en $(x_{0},y_{0})$, descrito por $$T={(x,y)\in\mathbb{R}^{2}~|~|x-x_{0}<\epsilon,|y-y_{0}|<\epsilon|}$$obtenemos la misma función pero con dominio restringido a un intervalo $(x_{0}-\delta,x_{0}+\delta)$ con $\delta0$ tal que para todo x, si $|x-x_{0}|<\delta$ entonces $|f(x)-f(x_{0})|<\epsilon$. Por tanto, f es continua en $x_{0}$.\Para probar que f es continua en x $\forall~x\in (x_{0}-h,x_{0}+h)$ tómese $x_{1}$ en $(x_{0}-h,x_{0}+h)$ con $x_{1}\neq x_{0}$ y un $\epsilon>0$ lo suficientemente pequeño para garantizar que el cuadrado $$U=\left\{(x,y)\in\mathbb{R}^{2}~|~|x-x_{1}<\epsilon,|y-y_{1}|<\epsilon|\right\}$$ centrado en $(x_{1},y_{1})$ y donde $y_{1}=f(x_{1})$ este totalmente contenido en el cuadrado original S, y ademas para todo x tal que $|x-x_{1}<\epsilon$, $x\in(x_{0}-h,x_{0}+h)$. Así, repitiendo el proceso para determinar f, ahora restringiendonos a las x que cumplen $|x-x_{1}<\epsilon$, encontramos que existe una $0<\delta_{1}<\epsilon$ tal que, para todo x, si $|x-x_{1}<\delta_{1}$ entonces $|f(x)-f(x_{1})<\epsilon$. lo cual quiere decir que f es continua en $x_{1}$. Por consiguiente, f es continua en $(x_{0}-h,x_{0}+h)$

Ahora probaremos que $y’$ es continua en $I=(x_{0}-h,x_{0}+h)$ con derivada
$$y’=-\frac{\frac{\partial F}{\partial x}}{\frac{\partial F}{\partial y}}$$

Demostración. Como F tiene parciales continuas en $x_{0}$ entonces F es diferenciable en $x_{0}$ por lo tanto
$$F((x_{0},y_{0})+(h_{1},h_{2}))=F(x_{0},y_{0})+\frac{\partial F}{\partial x}(x_{0},y_{0})h_{1}+\frac{\partial F}{\partial y}(x_{0},y_{0})h_{2}+r(h_{1},h_{2})$$donde
$$\lim_{(h_{1},h_{2})\rightarrow(0,0)}\frac{r(h_{1},h_{2})}{|(h_{1},h_{2})|}=0$$
tomando $x_{0}+h_{1}\in I$ y haciendo $y_{0}+h_{2}=f(x_{0}+h_{1})$ se tiene
$$F((x_{0},y_{0})+(h_{1},h_{2}))=F(x_{0}+h_{1},f(x_{0}+h_{2}))=0$$
también
$$F(x_{0},y_{0})=0$$por lo tanto
$$F(x_{0}+h_{1},f(x_{0}+h_{2}))-F(x_{0},y_{0})=0$$esto quiere decir
$$\frac{\partial F}{\partial x}(x_{0},y_{0})h_{1}+\frac{\partial F}{\partial y}(x_{0},y_{0})h_{2}+r(h_{1},h_{2})=0$$
como
$$r(h_{1},h_{2})=0parah_{1},h_{2}$$ cercanas a 0
$$\frac{\partial F}{\partial x}(x_{0},y_{0})h_{1}+\frac{\partial F}{\partial y}(x_{0},y_{0})h_{2}=0$$
por lo tanto
$$\frac{h_{2}}{h_{1}}=-\frac{\frac{\partial F}{\partial x}(x_{0},y_{0})}{\frac{\partial F}{\partial y}(x_{0},y_{0})}$$
pero $h_{2}=\triangle y$ y $h_{1}=\triangle x$ por lo tanto
$$\frac{\triangle y}{\triangle x}=-\frac{\frac{\partial F}{\partial x}(x_{0},y_{0})}{\frac{\partial F}{\partial y}(x_{0},y_{0})}$$
haciendo $\triangle y~\triangle x~\rightarrow~0$ se tiene
$$y'(x_{0})=\frac{dy}{dx}=-\frac{\frac{\partial F}{\partial x}(x_{0},y_{0})}{\frac{\partial F}{\partial y}(x_{0},y_{0})}$$
este mismo razonamiento es valido para $x\in I$. $\quad$

Teorema de la Función Implícita ( Versión (2))

Considere la función $F(x,y,z)$. Sea $(x_{0},y_{0},z_{0}) \in \mathbb{R}^{3}$ un punto tal que $F(x_{0},y_{0},z_{0})=0$. Suponga que la función F tiene derivadas parciales $\displaystyle{\frac{\partial F}{\partial x},~\frac{\partial F}{\partial y},~\frac{\partial F}{\partial z}}$ continuas en alguna bola con centro $(x_{0},y_{0},z_{0})$ y que $\displaystyle \frac{\partial F}{\partial z}(x_{0},y_{0},z_{0})\neq 0$.
Entonces $F(x,y,z)=0$ se puede resolver para $z$ en términos de $x,y$ y definir así una función $z=f(x,y)$ con dominio en una vecindad de $(x_{0},y_{0},z_{0})$, tal que $z_{0}=f(x_{0},y_{0})$, lo cual tiene derivadas continuas en $\mathcal{V}$ que pueden calcularse como $$\frac{d z}{dx}(x,y)=-\displaystyle \frac{\displaystyle \frac{\partial F}{\partial x}(x,y)}{\displaystyle \frac{\partial F}{\partial z}(x,y)}~~~\frac{d z}{dy}(x,y)=-\displaystyle \frac{\displaystyle \frac{\partial F}{\partial y}(x,y)}{\displaystyle \frac{\partial F}{\partial z}(x,y)}$$
$\textbf{Importante:}$ Este es un resultado que garantiza la existencia de una función $z=f(x,y)$ definida implícitamente por $F(x,y,z)=0$. Esto es, puede resolverse para $z$ en términos de $x,y$, pero no nos dice como hacer el despeje.

Ejemplo. Sea $f(x,y,z)=x+y+z-ze^{z}$ entonces $\displaystyle{\frac{\partial F}{\partial z}=1-e^{z}(z+1)}$ si el punto $P(x_{0},y_{0},z_{0}) \in \mathbb{R}^{3}$ es tal que $x_{0}+y_{0}+z_{0}e^{z_{0}}=0$ y $z\neq0$ y como $\displaystyle \frac{\partial F}{\partial z}\neq 0$. El $\textbf{T.F.Im.}$ sugiere que podamos despejar $z$ en términos de $x$ y $y$ y establecer así una función $z=f(x,y)$ con $z_{0}=f(x_{0},y_{0})$ de modo que su gráfica en los alrededores de $P$ coincide con $F(x,y,z)=0$. Las parciales de la función $f$ son

$\displaystyle \frac{\partial F}{\partial x}= \displaystyle \frac{\displaystyle \frac{-\partial F}{\partial x}}{\displaystyle \frac{\partial F}{\partial z
}}=\displaystyle \frac{-1}{1-e^{z}(z+1)}$,$~~~~$ $\displaystyle
\frac{\partial F}{\partial y}= \displaystyle \frac{\displaystyle
\frac{-\partial F}{\partial y}}{\displaystyle \frac{\partial
F}{\partial z }}=\displaystyle \frac{-1}{1-e^{z}(z+1)}$.

Ejercicio. Si $$\frac{d z}{dx}(x,y)=-\displaystyle \frac{\displaystyle \frac{\partial F}{\partial x}(x,y)}{\displaystyle \frac{\partial F}{\partial z}(x,y)}$$ calcular $$\frac{\partial^{2}F}{\partial x^{2}}$$

Solución. tenemos que
$$\frac{\partial^{2}F}{\partial x^{2}}=\frac{\partial}{\partial x}\left(-\displaystyle
\frac{\displaystyle \frac{\partial F}{\partial x}(x,y)}{\displaystyle \frac{\partial F}{\partial z}(x,y)}\right)=-\frac{\left( \frac{\partial F}{\partial z}\right)\left[ \frac{\partial^{2} F}{\partial x^{2}} \frac{dx}{d x}+ \frac{\partial^{2} F}{\partial y\partial x} \frac{dy}{dx}+ \frac{\partial^{2} F}{\partial z\partial x} \frac{dz}{dx}\right]-\left( \frac{\partial F}{\partial x}\right)\left[ \frac{\partial^{2} F}{\partial x \partial z}\frac{dx}{d x}+ \frac{\partial^{2} F}{\partial y\partial z} \frac{dy}{dx}+ \frac{\partial^{2} F}{\partial z^{2}} \frac{dz}{dx}\right]}{\left(\frac{\partial F}{\partial z}\right)^{2}}$$
$$=-\frac{\left( \frac{\partial F}{\partial z}\right)\left[ \frac{\partial^{2} F}{\partial x^{2}}+ \frac{\partial^{2} F}{\partial z\partial x} \frac{dz}{dx}\right]-\left( \frac{\partial F}{\partial x}\right)\left[ \frac{\partial^{2} F}{\partial x \partial z}+\frac{\partial^{2} F}{\partial z^{2}} \frac{dz}{dx}\right]}{\left(\frac{\partial F}{\partial z}\right)^{2}}$$
$$=-\frac{\left( \frac{\partial F}{\partial z}\right)\left[ \frac{\partial^{2} F}{\partial x^{2}}+ \frac{\partial^{2} F}{\partial z\partial x} \left(-\frac{\frac{\partial F}{\partial x}}{\frac{\partial F}{\partial z}}\right)\right]-\left( \frac{\partial F}{\partial x}\right)\left[ \frac{\partial^{2} F}{\partial x \partial z}+\frac{\partial^{2} F}{\partial z^{2}}\left(-\frac{\frac{\partial F}{\partial x}}{\frac{\partial F}{\partial z}}\right)\right]}{\left(\frac{\partial F}{\partial z}\right)^{2}}$$
$$=-\frac{\left( \frac{\partial F}{\partial z}\right)^{2} \frac{\partial^{2} F}{\partial x^{2}}-2 \frac{\partial^{2} F}{\partial z\partial x} \frac{\partial F}{\partial x}\frac{\partial F}{\partial z}+\left(\frac{\partial F}{\partial x}\right)^{2}{\frac{\partial^{2} F}{\partial z^{2}}}}{\left(\frac{\partial F}{\partial z}\right)^{3}}$$

Teorema de la Función Implícita (Versión (3))

Teorema 1. Considere la función $z=f(x_{1},…,x_{n})$. Sea $p=(x_{1},…,x_{n},y) \in \mathbb{R}^{n+1}$ un punto tal que $F(p)=0$. Suponga que la función $F$ tiene derivadas parciales $\displaystyle \frac{\partial F}{\partial x_{i}}$, $i=1,…,n$, y $\displaystyle \frac{\partial F}{\partial y}$ continuas en alguna bola con centro $P$ y que $\displaystyle \frac{\partial F}{\partial y}\neq 0$.
Entonces, $F(x_{1}$,…,$x_{n})=0$ puede resolverse para $y$ en términos de $x$ y definir así una vecindad $v$ de $\mathbb{R}^{n}$ del punto $(x_{1},$…,$x_{n})$, una función $y=f(x_{1}$,…,$x_{n})$ lo cual tiene derivadas parciales continuas en $v$ que se pueden calcular con las fórmulas $\displaystyle \frac{\partial F}{\partial x_{i}}(x_{1}$,…,$x_{n})=\displaystyle \frac{\displaystyle \frac{-\partial F}{\partial x_{i}}(x_{1},….,x_{n})}{\displaystyle \frac{\partial F}{\partial y}(x_{1},…,x_{n})}$ con $(x_{1},…,x_{n}) \in v$.

Demostración. Una idea de como probar lo anterior es la siguiente:
Como $\frac{\partial F}{\partial \textcolor{Red}{y}}\neq 0$ entonces tenemos que $\frac{\partial F}{\partial \textcolor{Red}{y}}> 0$ ó $\frac{\partial F}{\partial \textcolor{Red}{y}}<0$ supongamos sin perdida de generalidad que $\frac{\partial F}{\partial \textcolor{Red}{y}}> 0$ entonces tenemos que $F(x_{1},x_{2},…,x_{q},y)$ es creciente cuando $(x_{1},…,x_{q})$ es constante $F(a_{1},…,a_{q},\textcolor{Red}{y})$ es creciente $\forall y\in [b-\epsilon,b+\epsilon]$ además se tiene que $F(a_{1},…,a_{q},b)=0$ entonces $$F(a_{1},…,a_{q},b+\epsilon)>0\quad
F(a_{1},…,a_{q},b-\epsilon)<0$$ $\therefore$ Si $(x_{1},…,x_{q})\in B_{\delta}(a_{1},…,a_{q})$ entonces $$F(x_{1},…,x_{q},b+\epsilon)>0\quad
F(x_{1},…,x_{q},b-\epsilon)<0\quad y\quad F\quad continua$$ se
tiene entonces que $\exists !\quad \textcolor{Red}{y}=f(x_{1},…,x_{q})\in [b-\epsilon,b+\epsilon]$ tal que $F(x_{1},x_{2},…,x_{q},f(x_{1},x_{2},…,x_{q}))=0$ y
$b=f(x_{1},x_{2},…,x_{q})$. Hemos encontrado que si $(x_{1},…,x_{q})\in B_{\delta}(a_{1},…,a_{q})$ entonces $f(x_{1},…,x_{q})=\textcolor{Red}{y}\in (b-\epsilon,b+\epsilon)$
$\therefore$ f es continua. $\square$

Convergencia y diferenciación

Por Lizbeth Fernández Villegas

Introducción

En la entrada anterior vimos que cuando una sucesión de funciones continuas converge uniformemente, podemos concluir que el límite es también una función continua. ¿Qué ocurrirá con funciones diferenciables?

Considera el espacio de funciones con dominio en $[a,b]$ con $a,b$ e imagen en $\mathbb{R}.$ Tal vez intuimos que si tenemos una sucesión de funciones diferenciables $(f_n)_{n \in \mathbb{N}}$ que convergen uniformemente a una función $f$ en $[a,b]$ entonces $f$ también es diferenciable y la sucesión de derivadas $(f’_n)_{n \in \mathbb{N}}$ converge uniformemente en $f’.$ Esto es falso, como muestra el siguiente:

Ejemplo. La sucesión $\left( \dfrac{sen (nx)}{\sqrt{n}} \right) _{n \in \mathbb{N}}$

Para cada $n \in \mathbb{N}$ sea $f_n:[0,1] \to \mathbb{R} \,$ tal que $f_n(x)=\dfrac{sen (nx)}{\sqrt{n}}.$ Ocurre que $\left( \dfrac{sen (nx)}{\sqrt{n}} \right) _{n \in \mathbb{N}}$ converge uniformemente a la función $f(x)=0.$

Sucesión $\left( \dfrac{sen (nx)}{\sqrt{n}} \right) _{n \in \mathbb{N}}.$

Esto es porque, para cualquier $x \in [0,1], \, |sen(nx)|<1.$ Por otro lado, $\sqrt{n} \to \infty.$ Por lo tanto $\left|\dfrac{sen (nx)}{\sqrt{n}} \right| = \dfrac{|sen(nx)|}{\sqrt{n}} \leq \dfrac{1}{\sqrt{n}} \to 0.$

Por otro lado, para cada $n \in \mathbb{N}$ se tiene que $f'(x)= \sqrt{n} \, cos(nx).$ Pero $(f’_n)_{n \in \mathbb{N}} \,$ no converge a $f’$ ni de forma puntual. Por ejemplo $f’_n(0)=\sqrt{n}$ tiende a $\infty$ mientras que $f'(0)=0.$

Ejemplo. La sucesión $\left( \dfrac{x}{1 + n x^2} \right) _{n \in \mathbb{N}}$

Para cada $n \in \mathbb{N}$ sea $f_n:\mathbb{R} \to \mathbb{R}$ tal que $f_n(x)=\dfrac{x}{1 + n x^2}.$

Sucesión $\left( \dfrac{x}{1 + n x^2} \right) _{n \in \mathbb{N}}.$

Comencemos identificando la función límite $f$ de la sucesión $(f_n)_{n \in \mathbb{N}}$ y la función límite $g$ de la sucesión de derivadas $(f’_n)_{n \in \mathbb{N}}.$

Ya la imagen anterior nos induce a proponer $f=0.$ También podemos observar que cada función tiene máximo y mínimo global cuya distancia a $0$ coincide. Además, estos se van acercando más al eje horizontal a medida que avanzamos en las funciones de la sucesión.

En efecto, cuando la derivada es $0,$ la función $f_n$ alcanza su máximo o mínimo global:
$$\dfrac{1-nx^2}{(nx^2+1)^2}=0 \, \iff \, 1-nx^2 = 0 \, \iff \, x = \pm \sqrt{\frac{1}{n}}$$

Esto significa que cada $f_n$ está acotada como sigue:
$|f_n(x)|= \left| \dfrac{x}{1 + n x^2} \right| \leq \left|\dfrac{\sqrt{\frac{1}{n}}}{1 + n \sqrt{\frac{1}{n}}^2}\right| = \dfrac{1}{2\sqrt{n}} \, \to \, 0.$

Lo cual prueba que $(f_n)_{n \in \mathbb{N}}$ converge uniformemente a $0.$

Para el límite de la sucesión de derivadas veamos la siguiente imagen.

Esto incentiva proponer $g$ como:

\begin{equation*}
g(x) = \begin{cases}
0 & \text{si x $\neq$ 0} \\
1 & \text{si $x = 0$}
\end{cases}
\end{equation*}

Entonces $f’$ no coincide con $g,$ pues asignan valores diferentes al ser evaluadas en $0.$ Dejaremos como ejercicio lo siguiente:

  1. Probar que $(f’_n)_{n \in \mathbb{N}} \to g.$ ¿La convergencia es puntual o uniforme?
  2. Identifica para qué valores de $x \in \mathbb{R}$ sí se cumple que $f'(x)=g(x).$
  3. ¿En qué intervalos de $\mathbb{R}$ se da la convergencia uniforme de $(f_n)_{n \in \mathbb{N}}$ en $f.$
  4. ¿En qué intervalos de $\mathbb{R}$ se da la convergencia uniforme de $(f’_n)_{n \in \mathbb{N}}$ en $g.$

Ejemplo. La sucesión $\left( \dfrac{1}{n} \, e^{-n^2x^2} \right) _{n \in \mathbb{N}}$

Para cada $n \in \mathbb{N}$ sea $f_n:\mathbb{R} \to \mathbb{R}$ tal que $f_n(x)=\dfrac{1}{n} \, e^{-n^2x^2}.$

Sucesión $\left( \dfrac{1}{n} \, e^{-n^2x^2} \right) _{n \in \mathbb{N}}.$

Veamos que $(f_n)_{n \in \mathbb{N}}$ converge uniformemente en $\mathbb{R}$ a la función $f=0.$

Para cada $n \in \mathbb{N}$ y para cada $x \in \mathbb{R}, \, f'(x)= -2nxe^{-n^2x^2}.$ Se puede demostrar que esta función alcanza su máximo global cuando $f'(x)=0, \,$ lo cual ocurre cuando $x=0.$ Entonces el máximo de $f_n$ está dado por $f(0)= \frac{1}{n} \, \to \, 0.$ Por lo tanto $(f_n)_{n \in \mathbb{N}}$ converge uniformemente en $\mathbb{R}$ a la función $f=0.$

Ahora observemos la sucesión de derivadas $(f’_n)_{n \in \mathbb{N}}.$

Dejamos como ejercicio al lector probar que $(f’_n)_{n \in \mathbb{N}}$ converge puntualmente a la función $g=0.$ No obstante, esta convergencia no es uniforme en ningún intervalo que contenga al origen.

Habiendo visto estas situaciones, conozcamos algunas condiciones de convergencia para $(f_n)_{n \in \mathbb{N}} \,$ y para $(f’_n)_{n \in \mathbb{N}} \,$ que implican que $f’ =g.$

Proposición: Para cada $n \in \mathbb{N}$ sea $f_n:[a,b] \to \mathbb{R}$ continua y diferenciable en $[a,b],$ tal que la sucesión $(f_n)_{n \in \mathbb{N}}$ converge puntualmente a $f:[a,b] \to \mathbb{R}$ y la sucesión de derivadas $(f’_n)_{n \in \mathbb{N}}$ converge uniformemente a $g:[a,b] \to \mathbb{R}.$ Entonces $f$ es continua y diferenciable en $[a,b]$ y $f’=g.$

Demostración:
Sean $j,k \in \mathbb{N}$ y $x_0 \in (a,b).$ La función $f_j-f_k$ es continua y diferenciable en $[a,b],$ particularmente, para cada $x \in (a,b),$ también lo será en el intervalo $(x_0,x)$ (o $(x,x_0)$ dependiendo del orden de los puntos). Según el teorema del valor medio, que se puede consultar en Cálculo Diferencial e Integral I: Teorema de Rolle y teorema del valor medio, existe $\xi_x \in (x_0,x)$ tal que:

$$\frac{(f_j-f_k)(x)-(f_j-f_k)(x_0)}{x-x_0}=(f’_j-f’_k)(\xi_x)$$

Entonces
$$(f_j-f_k)(x)-(f_j-f_k)(x_0)=((f’_j-f’_k)(\xi_x))(x-x_0)$$
Y si desarrollamos vemos que
$$f_j(x)-f_j(x_0)-f_k(x)+f_k(x_0)=(f’_j(\xi_x)-f’_k(\xi_x))(x-x_0)$$
Así
\begin{align*}
|f_j(x)-f_j(x_0)-f_k(x)+f_k(x_0)|&=|(f’_j(\xi_x)-f’_k(\xi_x))(x-x_0)| \\
& \leq \norm{f’_j-f’_k}_\infty |x-x_0|
\end{align*}

Dado que $(f’_n)_{n \in \mathbb{N}}$ converge uniformemente en $\mathcal{C}^0[a,b],$ para cada $\varepsilon >0$ existe $N_1 \in \mathbb{N}$ tal que para cada $x \in (a,b)$ y para cada $j,k \geq N_1:$

\begin{align*}
|f_j(x)-f_j(x_0)-f_k(x)+f_k(x_0)|& \leq \norm{f’_j-f’_k}_\infty |x-x_0| \\
& \leq \frac{\varepsilon}{3}|x-x_0|.
\end{align*}
Haciendo $j \to \infty$ se sigue que
$$|f(x)-f(x_0)-f_k(x)+f_k(x_0)|\leq \frac{\varepsilon}{3}|x-x_0|.$$

Por otro lado, como $(f’_n(x_0))_{n \in \mathbb{N}} \to g(x_0)$ existe $N_2 \in \mathbb{N}$ tal que para cada $k \geq N_1, \, |f’_k(x_0) – g(x_0)|< \frac{\varepsilon}{3}$

Sea $N= máx \{ N_1,N_2 \}.$ Existe $\delta >0$ tal que si $|x – x_0| < \delta$ entonces
$$\left| \frac{f_N(x)-f_N(x_0)}{x-x_0}-f’_N(x_0) \right| <\frac{\varepsilon}{3}.$$

Finalmente aplicamos la desigualdad de triángulo para concluir que
\begin{align*}
\left| \frac{f(x)-f(x_0)}{x-x_0}-g(x_0) \right| &\leq \left| \frac{f(x)-f(x_0)}{x-x_0} – \frac{f_N(x)-f_N(x_0)}{x-x_0} \right| + \left| \frac{f_N(x)-f_N(x_0)}{x-x_0}-f’_N(x_0) \right|+ |f’_N(x_0) – g(x_0)|\\
&\leq \frac{\varepsilon}{3}+\frac{\varepsilon}{3}+\frac{\varepsilon}{3}\\
&= \varepsilon
\end{align*}

Por lo tanto $f$ es diferenciable en $x_0$ y $f'(x_0)=g(x_0).$ Ya que las derivadas $f’_n$ son continuas y convergen uniformemente se sigue por lo visto en la entrada anterior que $f$ es continuamente diferenciable.

Hay un resultado más fuerte sobre convergencia uniforme y diferenciación. La prueba de este se omite pero puede consultarse en Apostol, T., Análisis Matemático (2a ed.). México: Editorial Reverté, 1996. Pag 278. Se enuncia como sigue:

Proposición: Para cada $n \in \mathbb{N}$ sea $f_n:(a,b) \to \mathbb{R}.$ Supongamos que para un punto $x_0 \in (a,b)$ la sucesión $(f_n(x_0))_{n \in \mathbb{N}}$ converge. Supongamos además que la sucesión de derivadas $(f’_n)_{n \in \mathbb{N}}$ converge uniformemente en $(a,b)$ a una función $g.$ Entonces la sucesión $(f_n)_{n \in \mathbb{N}}$ converge uniformemente en $(a,b)$ a una función $f$ derivable en $(a,b)$ y $f’=g.$

Más adelante…

Conoceremos la relación entre una sucesión de funciones integrables con su función límite. ¿Bajo qué condiciones será también integrable?

Tarea moral

  1. Resuelve las actividades que quedaron pendientes en los ejemplos de esta entrada.

Enlaces:

Regla de la Cadena, Teorema de la función implícita

Por Angélica Amellali Mercado Aguilar

Regla de la Cadena

Ejemplo. Dadas $g(x,y)=(xy,5x,y^{3})$ y $f(x,y,z)=(3x^{2}+y^{2}+z^{2},5xyz)$.

Calcular $JF\circ g$

Demostración. En este caso

$$Jf(g)=\left(\begin{matrix}\frac{\partial f_{1}}{\partial x}(xy,5x,y^{3})&\frac{\partial f_{1}}{\partial y}(xy,5x,y^{3})&\frac{\partial f_{1}}{\partial z}(xy,5x,y^{3})\\ \frac{\partial f_{2}}{\partial x}(xy,5x,y^{3})&\frac{\partial f_{2}}{\partial y}(xy,5x,y^{3})&\frac{\partial f_{2}}{\partial z}(xy,5x,y^{3})\end{matrix}\right)=$$

$$\left(\begin{matrix}\frac{\partial (3x^{2}+y^{2}+z^{2})}{\partial x}(xy,5x,y^{3})&\frac{\partial (3x^{2}+y^{2}+z^{2})}{\partial y}(xy,5x,y^{3})&\frac{\partial (3x^{2}+y^{2}+z^{2})}{\partial z }(xy,5x,y^{3})\\ \frac{\partial (5xyz)}{\partial x}(xy,5x,y^{3})&\frac{\partial (5xyz)}{\partial y}(xy,5x,y^{3})&\frac{\partial (5xyz)}{\partial z}(xy,5x,y^{3})\end{matrix}\right)=$$

$$\left(\begin{matrix}6x\left|_{(xy,5x,y^{3})}\right.&2y\left|_{(xy,5x,y^{3})}\right.&2z\left|_{(xy,5x,y^{3})}\right.\\ 5yz\left|_{(xy,5x,y^{3})}\right.&5xz\left|_{(xy,5x,y^{3})}\right.&5xy\left|_{(xy,5x,y^{3})}\right.\end{matrix}\right)=\left(\begin{matrix}6xy&10x&2y^{3}\\ 25xy^{3}&5xy^{4}&25x^{2}y\end{matrix}\right)$$

Mientras que

$$Jg=\left(\begin{matrix}\frac{\partial g_{1}}{\partial x}&\frac{\partial g_{1}}{\partial y}\ \\ \frac{\partial g_{2}}{\partial x}&\frac{\partial g_{2}}{\partial y}\\ \frac{\partial g_{3}}{\partial x}&\frac{\partial g_{3}}{\partial y}\end{matrix}\right)=\left(\begin{matrix}\frac{\partial (xy)}{\partial x}&\frac{\partial (xy)}{\partial y}\\ \frac{\partial (5x)}{\partial x}&\frac{\partial (5x)}{\partial y}\ \frac{\partial (y^{3})}{\partial x}&\frac{\partial (y^{3})}{\partial y}\end{matrix}\right)=\left(\begin{matrix}y&x\\5&0\\0&3y^{2}\end{matrix}\right)$$

Por lo tanto

$$Jf\circ g=Jf(g)\cdot Jg=\left(\begin{matrix}6xy&10x&2y^{3}\\25xy^{3}&5xy^{4}&25x^{2}y\end{matrix}\right)\left(\begin{matrix}y&x\\5&0\\0&3y^{2}\end{matrix}\right)=\left(\begin{matrix}6xy^{2}+50x&6x^{2}y+6x^{5}\\50xy^{4}&100x^{2}y^{3}\end{matrix}\right)$$

Teorema 1. Sea $f:D’\subset \mathbb{R}^{m}\rightarrow \mathbb{R}^{p}$ una función definida en el abierto $D’\subset \mathbb{R}^{m}$ y sea $g:D\subset \mathbb{R}^{n}\rightarrow \mathbb{R}^{m}$ una función definida en el abierto $D\subset \mathbb{R}^{n}$. Si g es diferenciable en $x_{0}\in D$ y f es diferenciable en $g(x_{0})\in D’$ entonces la función $f\circ g:\mathbb{R}^{n}\rightarrow \mathbb{R}^{p}$ es diferenciable en $x_{0}$

Demostración. Tenemos que probar que

$\begin{equation}
\lim_{h\rightarrow0}\frac{|f(g(x_{0}+h))-f(g(x_{0}))-Jf(g(x_{0}))Jg(x_{0})h|}{|h|}=0
\end{equation}$

y para esto vamos a trabajar el numerador de la expresión anterior, tenemos entonces que

$$|f(g(x_{0}+h))-f(g(x_{0}))-Jf(g(x_{0}))Jg(x_{0})h|=$$
$$|f(g(x_{0}+h))-f(g(x_{0}))-Jf(g(x_{0}))(g(x_{0}+h)-gx_{0})+Jf(g(x_{0}))(g(x_{0}+h)-gx_{0})-Jf(g(x_{0}))Jg(x_{0})h|=$$
$$|f(g(x_{0}+h))-f(g(x_{0}))-Jf(g(x_{0}))(g(x_{0}+h)-gx_{0})+Jf(g(x_{0}))\left[(g(x_{0}+h)-gx_{0})-Jg(x_{0})h\right]|\leq$$
$$|f(g(x_{0}+h))-f(g(x_{0}))-Jf(g(x_{0}))(g(x_{0}+h)-gx_{0})|+|Jf(g(x_{0}))\left[(g(x_{0}+h)-gx_{0})-Jg(x_{0})h\right]|\leq$$

Como $|Jf(g(x_{0}))h|\leq M|h|$
$$|f(g(x_{0}+h))-f(g(x_{0}))-Jf(g(x_{0}))(g(x_{0}+h)-gx_{0})|+M|(g(x_{0}+h)-gx_{0})-Jg(x_{0})h|$$
Como g es diferenciable en $x_{0}$, dado $\epsilon>0$, existe $\delta_{1}>0$ tal que $|h|<\delta_{1}$ entonces
$$\frac{|(g(x_{0}+h)-gx_{0})-Jg(x_{0})h|}{|h|}<\frac{\epsilon}{2M}$$
por lo tanto
$$|(g(x_{0}+h)-gx_{0})-Jg(x_{0})h|<\frac{\epsilon|h|}{2M}$$
Ahora para
$$|f(g(x_{0}+h))-f(g(x_{0}))-Jf(g(x_{0}))(g(x_{0}+h)-g(x_{0})|$$
Como f es diferenciable en $g(x_{0})$ entonces
$$\frac{|f(g(x_{0}+h))-f(g(x_{0}))-Jf(g(x_{0}))h|}{|h|}<\frac{\epsilon}{2M_{1}}~\Rightarrow~|f(g(x_{0}+h))-f(g(x_{0}))-Jf(g(x_{0}))h|<\frac{\epsilon}{2M_{1}}|h|$$
por lo tanto
$$|f(g(x_{0}+h))-f(g(x_{0}))-Jf(g(x_{0}))(g(x_{0}+h)-g(x_{0})|<\frac{\epsilon}{2M_{1}}|g(x_{0}+h)-g(x_{0})|$$
ahora bien
$$|g(x_{0}+h)-g(x_{0})|=|g(x_{0}+h)-g(x_{0})-Jg(x_{0})h+Jg(x_{0})h|\leq |g(x_{0}+h)-g(x_{0})-Jg(x_{0})h|+|Jg(x_{0})h|\underbrace{\leq}_{\epsilon=1} |h|+M|h|=|h|M{1} $$
por lo tanto
$$|f(g(x_{0}+h))-f(g(x_{0}))-Jf(g(x_{0}))(g(x_{0}+h)-g(x_{0})|<\frac{\epsilon}{2M_{1}}|g(x_{0}+h)-g(x_{0})|\leq \frac{\epsilon}{2M_{1}}|h|M_{1}=\frac{\epsilon}{2}|h|$$
regresando ahora a (1) y tomando $\delta=\min{\delta_{1},\delta_{2},\delta_{3}}$ se tiene que si $|h|<\delta$
$$\frac{|f(g(x_{0}+h))-f(g(x_{0}))-Jf(g(x_{0}))Jg(x_{0})h|}{|h|}<\frac{1}{|h|}\left(\frac{\epsilon}{2}|h|+M\frac{\epsilon|h|}{2M}\right)=\epsilon$$
por lo tanto
$$\lim_{h\rightarrow0}\frac{|f(g(x_{0}+h))-f(g(x_{0}))-Jf(g(x_{0}))Jg(x_{0})h|}{|h|}=0$$

Teorema de la Función Implícita (versión 1)

Teorema 2. Considere la función $y=f(x)$. Sea $(x_{0},y_{0}) \in \mathbb{R}^{2}$ un punto tal que $F(x_{0},y_{0})=0$. Suponga que la función $F$ tiene derivadas parciales continuas en alguna bola con centro $(x_{0},y_{0})$ y que $\displaystyle \frac{\partial F}{\partial y}(x_{0},y_{0})\neq 0$.
Entonces $F(x,y)=0$ se puede resolver para $y$ en términos de $x$ y definir así una función $y=f(x)$ con dominio en una vecindad de $(x_{0},y_{0})$, tal que $y_{0}=f(x_{0})$, lo cual tiene derivadas continuas en $\mathcal{V}$ que pueden calcularse como $y’=f'(x)=-\displaystyle \frac{\displaystyle \frac{\partial F}{\partial
x}(x,y)}{\displaystyle \frac{\partial F}{\partial y}(x,y)}$, $x \in \mathcal{V}$.

Demostración. Como $\displaystyle{\frac{\partial
F}{\partial y}(x_{0},y_{0})\neq 0}$ supongamos sin perdida de generalidad que $\displaystyle{\frac{\partial
F}{\partial y}(x_{0},y_{0})> 0}$. Por ser $\displaystyle{\frac{\partial
F}{\partial y}}$ continua en una vecindad de $(x_{0},y_{0})$ entonces exite un cuadrado S, centrado en $(x_{0},y_{0})$ totalmente contenido en esa vecindad, en donde $\displaystyle{\frac{\partial
F}{\partial y}(x,y)> 0}$ $\forall~x,y\in S$. Sea
$$S=\left\{(x,y)\in\mathbb{R}^{2}~|~|x-x_{0}|<k~y~|y-y_{0}<k|\right\}$$

En todo punto $(x,y)$ que pertenece a S, $\displaystyle{\frac{\partial F}{\partial y}(x,y)>0}$. Esto quiere decir que en $S$,$F$ es creciente y fijando $x_{0}$ en $[x_{0}-h,x_{0}+h])$ se tiene que F es creciente en $[y_{0}-k,y_{0}+k]$ y se anula en $y_{0}$, por lo que
$$F(x_{0},y_{0}-k)<0~~y~~F(x_{0},y_{0}+k)>0$$

Consideremos ahora el par de funciones $F(x,y_{0}-k)$ y $F(x,y_{0}+k)$ definidas en el intervalo $(x_{0}-k,x_{0}+k)$. Donde ambas funciones solo tienen $x$ como variable. La primera función cumple $F(x_{0},y_{0}-k)<0$ y por ser continua en $x_{0}$, es negativa en toda una vecindad $(x_{0}-h_{1}x_{0}+h_{1})$ de $x_{0}$. Análogamente, la segunda función cumple $F(x_{0},y_{0}+k)>0$ y por ser continua en $x_{0}$, es positiva en toda una vecindad $(x_{0}-h_{2}x_{0}+h_{2})$ de $x_{0}$. Sea $h=\min \left\{h_{1},h_{2}\right\}$. Entonces para toda $x$ tal que $$|x-x_{0}|~y~F(x,y_{0}+k)>0$$
Fijemos $x$ en el intervalo $(x_{0}-h,x_{0}+h)$, y consideremos a $F(x,y)$, sólo como función de y, sobre $[y_{0}-k,y_{0}+k]$. Esta función cumple que
$$F(x,y_{0}-k)<0~y~F(x,y_{0}+k)>0$$ por lo tanto segun el teorema del valor intermedio, existe un único y en $(y_{0}-k,y_{0}+k)$ tal que $F(x,y)=0$. Así queda establecida la existencia y unicidad de la función $y=f(x)$. Donde además, $y_{0}=f(x_{0})$, y para todo $x\in(x_{0}-h,x_{0}+h)$
$$F(x,f(x))=0,~~y~~\frac{\partial
F}{\partial y}(x_{0},y_{0})\neq 0$$ $\quad$


Teorema de la función inversa

Por Angélica Amellali Mercado Aguilar

Introducción

Teorema de la Función Inversa $f:\mathbb{R}\rightarrow\mathbb{R}$

Teorema 1. Sea $f:A\subset\mathbb{R}\rightarrow\mathbb{R}$ definida sobre el abierto $A\subset\mathbb{R}$ y sea $x_{0}\in A$.
(1) Supóngase que f tiene derivada continua y que $f'(x_{0})\neq 0$.
(2) Entonces existe un intervalo abierto $I$ que contiene al punto $x_{0}$ y un intervalo abierto $j$ que contiene a $f(x_{0})$, tal que la función $f:I\rightarrow J$ es uno a uno y sobre.
(3) Además, la función inversa $f^{-1}:J\rightarrow I$ también tiene derivada continua y para un punto $y\in J$, si $x\in I$ es tal que $f(x)=y$, entonces
$$\left(f^{-1}\right)'(y)=\frac{1}{f'(x)}$$

Ejercicio. Obtener la tesis del teorema de la función inversa como aplicación del teorema de la función implícita

Solución. Sea $y=f(x)$ una función real de variable real con derivada continua sobre un conjunto abierto A y sea $x_{0}$ un punto de A donde $f'(x_{0})\neq0$.

Considere la función $F(x,y)=y-f(x)$ y calculemos sus derivadas parciales. Así
$$\frac{\partial F}{\partial x}=-f'(x)~~y\frac{\partial F}{\partial y}=1$$ Nótese que $F,\displaystyle{\frac{\partial F}{\partial x},~~\frac{\partial F}{\partial y}}$ son continuas sobre el conjunto
$$B=\left\{(x,y)\in\mathbb{R}^{2}~|~x\in A \right\}$$

Considere ahora como solución inicial el punto $(x_{0},y_{0})$ donde $y_{0}=f(x_{0})$. Tenemos que
$$F(x_{0},y_{0})=0~~y\frac{\partial F}{\partial x}(x_{0},y_{0})\neq0$$ De manera que se cumplen las hipótesis del Teorema de la Función Implicita. Luego entonces cerca del punto $(x_{0},y_{0})$ la variable x puede representarse en términos de la variable y. Estos expresado formalmente nos dice que existe una única función implicita $x=g(y)$ con dominio un intervalo $J=(y_{0}-k,y_{0}+k)$ y con rango $I=(x_{0}-h,x_{0}+h)$ tal que $$g(y_{0})=x_{0}$$ y, para toda y, en el intervalo J $$F(g(y),y)=0~y~~~\frac{\partial F}{\partial x}(g(y),y)\neq 0$$
ademas, g y su derivada $g’$ son continuas sobre J, y
$$g'(y)=-\frac{\frac{\partial F}{\partial y}(g(y),y)}{\frac{\partial F}{\partial x}(g(y),y)}=-\frac{1}{-f'(g(y))}=\frac{1}{f'(x)}$$La función g que ha sido determinada no es otra que la función inversa

Ejemplo. Sea f la función definida por la regla de correspondencia $f(x)=-x^{5}-x$. Si calculamos su derivada, tenemos $f'(x)=-5x^{4}-1$. Observese que $f'(x)<0$ para toda x en los reales, por lo que f es decreciente sobre toda la recta real y a su vez es uno a uno.\Concluimos así que la inversa de f está definida sobre toda la recta real y que su gráfica es decreciente. Sin embargo, no se puede obtener la regla de correspondencia para la inversa. Sin embargo, podemos calcular su derivada. Sea y cualquier número real y supóngase que x es tal que $f^{-1}(y)=x$. Así
$$\left(f^{-1}\right)'(y)=\frac{1}{f'(f^{-1}(y))}=\frac{1}{f'(x)}=-\frac{1}{5x^{4}+1}$$

Teorema de la Función Inversa (sistema $f_{i}:\mathbb{R}^{n}\rightarrow\mathbb{R}$)

Sea $U\subset\mathbb{R}^{n}$ un abierto y sean
$$\begin{matrix}
f_{1}:U\rightarrow\mathbb{R} \\
\vdots \\
f_{n}:U\rightarrow\mathbb{R}
\end{matrix}$$
con derivadas parciales continuas. Considerar las ecuaciones

$f_1(x_1,x_2,…,x_n)= y_1$
$f_2(x_1,x_2,…,x_n)= y_2$
$\vdots\\$
$f_n(x_1,x_2,…,x_n)= y_n$
Tratamos de resolver las n-ecuaciones para $x_1,x_2,… x_n$como funciones de $y_1,y_2,… y_n$

La condición de existencia para la solución en una vecindad del punto $x_0$ es que el determinante de la matriz $Df(x_0)$ y $f=(f_i,f_2,… f_n)$ sean distintos de cero. Explícitamente:

$\displaystyle \frac{\partial(f_1,f_2,…,f_n)}{\partial(x_1,x_2,…,x_n)}|_{x=x_0}= J(f)(x_0)= \left| \begin{array}{ccc} \displaystyle\frac{\partial f_1}{\partial x_1} (x_0)&\ldots&\displaystyle\frac{\partial f_1}{\partial x_1}(x_0)\\ \vdots & & \vdots\\
\displaystyle\frac{\partial f_n}{\partial x_1}(x_0)&\ldots&\displaystyle\frac{\partial f_n}{\partial x_n}(x_0) \end{array}\right|\neq 0$

entonces el sistema anterior se puede resolver de manera ‘unica como $x=g(y)$ para $x$ cerca de $x_{0}$ y y cerca de $y_{0}$

Nota: La cuestión de existencia se responde por medio del teorema general de la función implícita aplicado a las funciones $y_i-f_i(x_1,x_2,…,x_n)$ con las incognitas $x_1,x_2,…,x_n$.

Solución. Aquí las funciones son
$$u(x,y)=f_{1}(x,y)=\frac{x^{4}+y^{4}}{x},~~y~~v(x,y)=f_{2}(x,y)= sen~x+\cos~y$$
De acuerdo al teorema de la función inversa
$$\frac{\partial(f_{1},f_{2})}{\partial(x,y)}=\left|\begin{matrix}
\frac{\partial f_{1}}{\partial x} & \frac{\partial f_{1}}{\partial y} \\
\frac{\partial f_{2}}{\partial x} & \frac{\partial f_{2}}{\partial y}
\end{matrix}\right|$$
$$=\left|\begin{matrix}
\frac{3x^{4}-y^{4}}{x^{2}} & \frac{4y^{3}}{x} \\
\cos~x & -sen~y
\end{matrix}\right|= \frac{sen~y}{x^{2}}(y^{4}-3x^{4})-\frac{4y^{3}}{x}\cos~x$$
por lo tanto, en los puntos donde la expresión anterior no se anula, se puede resolver para $x$,$y$ en términos de $u$ y $v$.

Mas aún, si consideramos las expresiones:
$G(x,y,u,v)=x-f(u,v)=0$
$H(x,y,u,v)=y-g(u,v)=0$

Lo que pretendemos es «despejar» de ella a $u$ y $v$ en términos de $x$ e $y$ y poder establecer así las funciones $u=\varphi(x,y), v=\psi(x,y)$. Entonces el T.F.Im. (tercera versión) nos da las condiciones para que podamos hacer esto. Sea $P (x,y,u,v)\epsilon\mathbb{R}^4$ un punto tal que $G(p)=H(p)=0$. Supongamos que en una bola de centro en P las derivadas parciales de $G$ y $H$ son continuas. Si el jacobiano.

$$\displaystyle\frac{\partial(G,H)}{\partial(u,v)}=\left|\begin{matrix}
\frac{\partial G}{\partial u} & \frac{\partial G}{\partial v} \\
\frac{\partial H}{\partial u} & \frac{\partial H}{\partial v}
\end{matrix}\right|=\left|\begin{matrix}
-\frac{\partial f}{\partial u} & -\frac{\partial f}{\partial v} \\
-\frac{\partial g}{\partial u} & -\frac{\partial g}{\partial v}
\end{matrix}\right|=\left|\begin{matrix}
\frac{\partial f}{\partial u} & \frac{\partial f}{\partial v} \\
\frac{\partial g}{\partial u} & \frac{\partial g}{\partial v}
\end{matrix}\right|\neq0$$

en $P$, entonces es posible «despejar» de ellas a $u$ y $v$ en terminos de $x$ e $y$, y establecer así funciones $u=\varphi(x,y), v=\varphi(x,y)$ definidas en una vecindad $V$ de $(x,y)=F(u,v)$, las cuales tienen derivadas parciales continuas en $V$ que se pueden calcular como

$\displaystyle\frac{\partial G}{\partial u}=-\displaystyle\frac{\partial f}{\partial u} ~,~ \displaystyle\frac{\partial G}{\partial v}=-\displaystyle\frac{\partial f}{\partial v} ~,~ \displaystyle\frac{\partial H}{\partial u}=-\displaystyle\frac{\partial g}{\partial u} ~,~ \displaystyle\frac{\partial H}{\partial v}=-\displaystyle\frac{\partial g}{\partial v}$

$$\displaystyle\frac{\partial u}{\partial x}=-\displaystyle\frac{\displaystyle\frac{\partial(G,H)}{\partial((x,v)}}{\displaystyle\frac{\partial(G,H)}{\partial(u,v)}}=-\displaystyle\frac{1}{\displaystyle\frac{\partial(f,g)}{\partial(u,v)}}det
\left| \begin{array}{cc}
\displaystyle\frac{\partial G}{\partial x} & \displaystyle\frac{\partial G}{\partial v}\\
\displaystyle\frac{\partial H}{\partial x} & \displaystyle\frac{\partial H}{\partial v}
\end{array} \right|= -\displaystyle\frac{1}{\displaystyle\frac{\partial(f,g)}{\partial(u,v)}}det \left|\begin{array}{cc}
1 & -\displaystyle\frac{\partial f}{\partial v}\\
0 & -\displaystyle\frac{\partial g}{\partial v}\end{array}\right|=\displaystyle\frac{\displaystyle\frac{\partial g}{\partial v}}{\displaystyle\frac{\partial(f,g)}{\partial(u,v)}}
$$

Por lo tanto: $\displaystyle\frac{\partial u}{\partial x} = \displaystyle\frac{\displaystyle\frac{\partial g}{\partial v}}{\displaystyle\frac{\partial(f,g)}{\partial(u,v)}}$

$$\displaystyle\frac{\partial u}{\partial y}=-\displaystyle\frac{\displaystyle\frac{\partial(G,H)}{\partial((y,v)}}{\displaystyle\frac{\partial(G,H)}{\partial(u,v)}}=-\displaystyle\frac{1}{\displaystyle\frac{\partial(f,g)}{\partial(u,v)}}det
\left| \begin{array}{cc}
\displaystyle\frac{\partial G}{\partial y} & \displaystyle\frac{\partial G}{\partial v}\\
\displaystyle\frac{\partial H}{\partial y} & \displaystyle\frac{\partial H}{\partial v}
\end{array} \right|= -\displaystyle\frac{1}{\displaystyle\frac{\partial(f,g)}{\partial(u,v)}}det \left|\begin{array}{cc}
0 & -\displaystyle\frac{\partial f}{\partial v}\\
1& -\displaystyle\frac{\partial g}{\partial v}\end{array}\right|=-\displaystyle\frac{\displaystyle\frac{\partial f}{\partial v}}{\displaystyle\frac{\partial(f,g)}{\partial(u,v)}}
$$

Por lo tanto: $\displaystyle\frac{\partial u}{\partial y} = -\displaystyle\frac{\displaystyle\frac{\partial f}{\partial v}}{\displaystyle\frac{\partial(f,g)}{\partial(u,v)}}$

$$\displaystyle\frac{\partial v}{\partial x}=-\displaystyle\frac{\displaystyle\frac{\partial(G,H)}{\partial((u,x)}}{\displaystyle\frac{\partial(G,H)}{\partial(u,v)}}=-\displaystyle\frac{1}{\displaystyle\frac{\partial(f,g)}{\partial(u,v)}}det
\left| \begin{array}{cc}
\displaystyle\frac{\partial G}{\partial u} & \displaystyle\frac{\partial G}{\partial x}\\
\displaystyle\frac{\partial H}{\partial u} & \displaystyle\frac{\partial H}{\partial x}
\end{array} \right|= -\displaystyle\frac{1}{\displaystyle\frac{\partial(f,g)}{\partial(u,v)}}det \left|\begin{array}{cc}
-\displaystyle\frac{\partial f}{\partial u} & 1\\
-\displaystyle\frac{\partial g}{\partial u} & 0 \end{array}\right|=-\displaystyle\frac{\displaystyle\frac{\partial g}{\partial u}}{\displaystyle\frac{\partial(f,g)}{\partial(u,v)}}$$

$$\displaystyle\frac{\partial v}{\partial y}=-\displaystyle\frac{\displaystyle\frac{\partial(G,H)}{\partial((u,y)}}{\displaystyle\frac{\partial(G,H)}{\partial(u,v)}}=-\displaystyle\frac{1}{\displaystyle\frac{\partial(f,g)}{\partial(u,v)}}det
\left| \begin{array}{cc}
\displaystyle\frac{\partial G}{\partial u} & \displaystyle\frac{\partial G}{\partial y}\\
\displaystyle\frac{\partial H}{\partial u} & \displaystyle\frac{\partial H}{\partial y}
\end{array} \right|= -\displaystyle\frac{1}{\displaystyle\frac{\partial(f,g)}{\partial(u,v)}}det \left|\begin{array}{cc}
-\displaystyle\frac{\partial f}{\partial u} & 0\\
-\displaystyle\frac{\partial g}{\partial u} & 1 \end{array}\right|=\displaystyle\frac{\displaystyle\frac{\partial f}{\partial u}}{\displaystyle\frac{\partial(f,g)}{\partial(u,v)}}$$

En resumen tenemos: Sean $f,g: U \subseteq \mathbb{R}^2 \rightarrow \mathbb{R}$ funciones definidas en el conjunto abierto $U$ de $\mathbb{R}^2$. Sean $x=f(u,v)$, $y=g(u,v)$.

Suponga que alguna bola $B$ de $\mathbb{R}^2$ con centro $(u,v)$, las derivadas parciales $\displaystyle\frac{\partial f}{\partial u}$, $\displaystyle\frac{\partial f}{\partial v}$, $\displaystyle\frac{\partial g}{\partial u}$, $\displaystyle\frac{\partial g}{\partial v}$ son continuas.

Si el jacobiano $\displaystyle\frac{\partial(f,g)}{\partial (u,v)}$ es no nulo en $(u,v)$ entonces $\exists$ una vecindad $V$ de $\bar{x},\bar{y}$ donde podemos definir «funciones inversas» $u=\varphi(x,y),v=\psi(x,y)$ es decir tales que $$u=\varphi(x,y), v=\psi(x,y)$, y $f(\varphi(x,y), \psi(x,y))=x ,g(\varphi(x,y), \psi(x,y))=y$$

para $(x,y) \epsilon V$ las cuales tienen derivadas parciales continuas en $V$ que se calculan como

$$\displaystyle\frac{\partial u}{\partial x} = \displaystyle\frac{\displaystyle\frac{\partial g}{\partial v}}{\displaystyle\frac{\partial(f,g)}{\partial(u,v)}}\ ~,~ \displaystyle\frac{\partial u}{\partial y} = -\displaystyle\frac{\displaystyle\frac{\partial f}{\partial v}}{\displaystyle\frac{\partial(f,g)}{\partial(u,v)}} ~,~ \displaystyle\frac{\partial v}{\partial x}=-\displaystyle\frac{\displaystyle\frac{\partial g}{\partial u}}{\displaystyle\frac{\partial(f,g)}{\partial(u,v)}} ~,~ \displaystyle\frac{\partial v}{\partial y}=\displaystyle\frac{\displaystyle\frac{\partial f}{\partial u}}{\displaystyle\frac{\partial(f,g)}{\partial(u,v)}} \ast$$

Ahora bien con las funciones $u=\varphi(x,y),~~v=\psi(x,y)$. Podemos formar el sistema
$$\begin{matrix}G(x,y,u,v)=u-\varphi(x,y)\\H(x,y,u,v)=v-\psi(x,y)\end{matrix}$$
se tiene entonces que

$$\frac{\partial (G,H)}{\partial (x,y)}=\left|\begin{matrix}
\frac{\partial G}{\partial x} & \frac{\partial G}{\partial y} \\
\frac{\partial H}{\partial x} & \frac{\partial H}{\partial y}
\end{matrix}\right|$$

$$=\left|\begin{matrix}
-\frac{\partial \varphi}{\partial x} & -\frac{\partial \varphi}{\partial y} \\
-\frac{\partial \psi}{\partial x} & -\frac{\partial \psi}{\partial y}
\end{matrix}\right|$$
$$=\left|\begin{matrix}
\frac{\partial u}{\partial x} & \frac{\partial u}{\partial y} \\
\frac{\partial v}{\partial x} &\frac{\partial v}{\partial y}
\end{matrix}\right|$$
$$
JF^{-1}=\left[\begin{array}{cc}
\displaystyle\frac{\partial u}{\partial x} & \displaystyle\frac{\partial u}{\partial y}\\
\displaystyle\frac{\partial v}{\partial x} & \displaystyle\frac{\partial v}{\partial y} \end{array} \right]
$$

El resultado anterior $\ast$ nos dice como calcular las derivadas parciales $\displaystyle\frac{\partial u}{\partial x}$, $\displaystyle\frac{\partial u}{\partial y}$, $\displaystyle\frac{\partial v}{\partial x}$, $\displaystyle\frac{\partial v}{\partial y}$ en una vecindad $V$ de $(x,y)$ al sustituir las fórmulas correspondientes en $JF^{-1}$, recordando que $\displaystyle\frac{\partial(f,g)}{\partial(u,v)}= det(JF)$.

$$ JF^{-1}= \left|\begin{array}{cc}
\displaystyle\frac{\displaystyle\frac{\partial g}{\partial v}}{det(JF)} & -\displaystyle\frac{\displaystyle\frac{\partial g}{\partial u}}{det(JF)}\\
-\displaystyle\frac{\displaystyle\frac{\partial f}{\partial v}}{det(JF)} & \displaystyle\frac{\displaystyle\frac{\partial f}{\partial U}}{det(JF)}\end{array}\right|=\displaystyle\frac{1}{det(JF)} \left|\begin{array}{cc}
\displaystyle\frac{\partial g}{\partial v} & -\displaystyle\frac{\partial f}{\partial v}\\
-\displaystyle\frac{\partial g}{\partial u} & \displaystyle\frac{\partial f}{\partial u} \end{array}\right|
$$

Multipliquemos $JF$ y $JF^{-1}$, se obtiene

$$
(JF)(JF^{-1})= \left|\begin{array}{cc}
\displaystyle\frac{\partial f}{\partial u} & \displaystyle\frac{\partial f}{\partial v}\\
\displaystyle\frac{\partial g}{\partial u} & \displaystyle\frac{\partial g}{\partial v} \end{array}\right| \displaystyle\frac{1}{det(JF)}\left[\begin{array}{cc}
\displaystyle\frac{\partial g}{\partial v} & -\displaystyle\frac{\partial f}{\partial v}\\
-\displaystyle\frac{\partial g}{\partial u} & \displaystyle\frac{\partial f}{\partial u} \end{array}\right]= \displaystyle\frac{1}{det(JF)}\left|\begin{array}{cc}
\displaystyle\frac{\partial f}{\partial u} & \displaystyle\frac{\partial f}{\partial v}\\
\displaystyle\frac{\partial g}{\partial u} & \displaystyle\frac{\partial g}{\partial v} \end{array}\right| \left|\begin{array}{cc}
\displaystyle\frac{\partial g}{\partial v} & -\displaystyle\frac{\partial f}{\partial v}\\
-\displaystyle\frac{\partial g}{\partial u} & \displaystyle\frac{\partial f}{\partial u} \end{array}\right|
$$

$$= \displaystyle\frac{1}{det(JF)}\left|\begin{array}{cc} \displaystyle\frac{\partial f}{\partial u} \displaystyle\frac{\partial g}{\partial v} – \displaystyle\frac{\partial f}{\partial v} \displaystyle\frac{\partial g}{\partial u} & 0 \\
0& -\displaystyle\frac{\partial g}{\partial u} \displaystyle\frac{\partial f}{\partial v} + \displaystyle\frac{\partial g}{\partial v} \displaystyle\frac{\partial f}{\partial u} \end{array}\right|=\displaystyle\frac{\displaystyle\frac{\partial f}{\partial u}\displaystyle\frac{\partial g}{\partial v} – \displaystyle\frac{\partial f}{\partial v}\displaystyle\frac{\partial g}{\partial u}}{det(JF)}\left[\begin{array}{cc}
1 & 0\\
0 & 1 \end{array}\right] = \left[\begin{array}{cc}
1 & 0\\
0 & 1 \end{array}\right]
$$

Así concluimos que la matriz jacobiana de la función inversa de F es justamente la inversa de la matriz jacobiana de F. Es decir se tiene
$$JF^{-1}=(JF)^{-1}$$

Ejemplo. Considere las ecuaciones dadas por $x=u^2+v^3,~~y=u^2+uv$. Se tiene
que en $p=(1,2)$ $x=9,~y=3$.\
Las derivadas parciales de las funciones
$x=f(u,v)=u^3+v^3$ , $y=g(u,v)=u^2+uv$ son
$$
\displaystyle\frac{\partial f}{\partial u}=3u^2~,~ \displaystyle\frac{\partial f}{\partial v}=3v^2 ~,~ \displaystyle\frac{\partial g}{\partial u}=2u+v ~,~\displaystyle\frac{\partial g}{\partial v}=u
$$

La matriz jacobiana de f es
$$JF=\left|\begin{array}{cc}
\displaystyle\frac{\partial f}{\partial u} & \displaystyle\frac{\partial f}{\partial v}\\
\displaystyle\frac{\partial g}{\partial u} & \displaystyle\frac{\partial g}{\partial v} \end{array}\right|=\left|\begin{array}{cc}
3u^2 & 3v^2\
2u+v & u \end{array}\right|
$$
la cual en el punto $(1,2)$ es invertible pues
$$ detJF(1,2) = \left|\begin{array}{cc}
3 & 12\
4 & 1 \end{array}\right| = -45\neq0
$$

Así podemos concluir que en una bola $B’$ de $(9,3)$ se da la inversa $F^{-1}$ de $F$ o bien, que podemos despejar de $x=u^3+v^3 , y=u^2+uv$ a $u,v$ como funciones de $x$ e $y$, la cual es de clase $C^1$ en $B’$ y que su derivada es
$$JF^{-1}(x,y)= [JF(u,v)]^{-1}=\displaystyle\frac{1}{detJF} \left[\begin{array}{cc}
\displaystyle\frac{\partial g}{\partial v} & -\displaystyle\frac{\partial f}{\partial v}\\
-\displaystyle\frac{\partial g}{\partial u} & \displaystyle\frac{\partial f}{\partial u} \end{array}\right]=\displaystyle\frac{1}{3u^3-6uv^2-3v^3} \left|\begin{array}{cc}
u & -3v^2\
-(2u+v) & 3u^2 \end{array}\right|
$$
donde $x=u^3+v^3 , y=u^2+uv$. Es decir

$$\displaystyle\frac{\partial u}{\partial x}(u^3+v^3, u^2+uv)= \displaystyle\frac{u}{3u^3-6uv^2-3v^3}$$
$$\displaystyle\frac{\partial u}{\partial y}(u^3+v^3, u^2+uv)= \displaystyle\frac{-3v^2}{3u^3-6uv^2-3v^3}$$
$$\displaystyle\frac{\partial u}{\partial y}(u^3+v^3, u^2+uv)= \displaystyle\frac{-2u+v}{3u^3-6uv^2-3v^3}$$
$$\displaystyle\frac{\partial u}{\partial y}(u^3+v^3, u^2+uv)= \displaystyle\frac{3u^2}{3u^3-6uv^2-3v^3}$$

Considere las ecuaciones
$$x=u+v+e^{w}$$
$$y=u+w+e^{2v}$$
$$x=v+w+e^{3u}$$
para $p=(u,v,w)=(0,0,0)$ se tiene que $q=(x,y,z)=(1,1,1)$ el
determinante de la matriz jacobiana de la función $F(u,v,w))(x,y,z)$
es:

$$
det JF=\frac{\partial(x,y,z)}{\partial(u,v,w)}=\left|\begin{array}{ccc}
\frac{\partial x}{\partial u} & \frac{\partial x}{\partial v} & \frac{\partial x}{\partial w}\\
\frac{\partial y}{\partial u} & \frac{\partial x}{\partial v} &
\frac{\partial y}{\partial w}\\ \frac{\partial z}{\partial u} &
\frac{\partial z}{\partial v} & \frac{\partial z}{\partial
w}\end{array}\right|=\left|\begin{array}{ccc}
1 & 1 & e^{w}\\
1 & 2e^{2v} & 1\\ 3e^{3u} & 1 &
1\end{array}\right|_{(0,0,0)}=\left|\begin{array}{ccc}
1& 1 &1\\
1 & 2 &1\\3 & 1 & 1\end{array}\right|
$$

Si calculamos su determinante obtenemos

$$=\left|\begin{array}{ccc}
\textcolor{Green}{1}&\textcolor{Red} {1} &\textcolor{Blue}{1}\\
1 & 2 &1\\ 3 & 1 & 1\end{array}\right|=\textcolor{Green}{1\times
\left|\begin{array}{cc}
2 & 1 \\
1 & 1\end{array}\right|}-\textcolor{Red}{1\times
\left|\begin{array}{cc}
1 & 1 \\
3 & 1\end{array}\right|}+\textcolor{Blue}{1\times
\left|\begin{array}{cc}
1 & 2 \\
3 & 1
1\end{array}\right|}=\textcolor{Green}{1\times(2-1)}-\textcolor{Red}{1\times(1-3)}+\textcolor{Blue}{1\times(1-6)}=\textcolor{Green}{1}+\textcolor{Red}{2}-\textcolor{Blue}{5}=-2\neq
0$$

$\therefore$ Podemos localmente invertir la función $F$, entorno al punto $q$, donde podemos definir funciones de clase

$c^{1}$ $u(x,y,z), v(x,y,z)$ y $w(x,y,z)$. Ahora bien como $$JF^{-1}(q)=\left[JF(p)\right]^{-1}=\left|\begin{array}{ccc}
1& 1 &1\\
1 & 2 &1\\3 & 1 &
1\end{array}\right|^{-1}\underbrace{=}_{*}\left|\begin{array}{ccc}
-\frac{1}{2}& 0 &\frac{1}{2}\\
-1 & 1 &0\\ \frac{5}{2} & -1 & -\frac{1}{2}\end{array}\right|$$

  • Vamos a calcular la inversa usando la matriz de cofactores de la matriz $\left(\begin{array}{ccc}
    \textcolor{OliveGreen}{1}&\textcolor{Magenta} {1} &\textcolor{Violet}{1}\\
    \textcolor{Brown}{1} &\textcolor{Orange} {2}
    &\textcolor{Blue}{1}\\ \textcolor{RedViolet}{3} &
    \textcolor{Fuchsia}{1} &\textcolor{Emerald} {1}\end{array}\right)$

$$\left(\begin{array}{ccc}
\textcolor{OliveGreen}{(-1)^{1+1}\times \left|\begin{array}{cc}
2 & 1 \\
1 & 1\end{array}\right|}&\textcolor{Magenta} {(-1)^{1+2}\times
\left|\begin{array}{cc}
1 & 1 \\
3 & 1\end{array}\right|} &\textcolor{Violet}{(-1)^{1+3}\times
\left|\begin{array}{cc}
1 & 2 \\
3 & 1
1\end{array}\right|}\\
\textcolor{Brown}{(-1)^{2+1}\times \left|\begin{array}{cc}
1 & 1 \\
1 & 1\end{array}\right|} &\textcolor{Orange} {(1)^{2+2}\times
\left|\begin{array}{cc}
1 & 1 \\
3 & 1\end{array}\right|} &\textcolor{Blue}{(-1)^{2+3}\times
\left|\begin{array}{cc}
1 & 1 \\
3 & 1\end{array}\right|}\\ \textcolor{RedViolet}{(-1)^{3+1}\times
\left|\begin{array}{cc}
1 & 1 \\
2 & 1\end{array}\right|} & \textcolor{Fuchsia}{(-1)^{3+2}\times
\left|\begin{array}{cc}
1 & 1 \\
1 & 1\end{array}\right|} &\textcolor{Emerald} {(1)^{3+3}\times
\left|\begin{array}{cc}
1 & 1 \\
1 & 2\end{array}\right|}\end{array}\right)=\left(\begin{array}{ccc}
\textcolor{OliveGreen}{1}&\textcolor{Magenta} {2} & \textcolor{Violet}{-5}\\
\textcolor{Brown}{0} &\textcolor{Orange} {-2}
&\textcolor{Blue}{2}\\ \textcolor{RedViolet}{-1} &
\textcolor{Fuchsia}{0} &\textcolor{Emerald} {1}\end{array}\right)$$

Transponiendo la ultima matriz tenemos
$$\left(\begin{array}{ccc}
\textcolor{OliveGreen}{1}&\textcolor{Brown} {0} &\textcolor{RedViolet}{-1}\\
\textcolor{Magenta}{2} &\textcolor{Orange} {-2}
&\textcolor{Fuchsia}{0}\\ \textcolor{Violet}{-5} &
\textcolor{Blue}{2} &\textcolor{Emerald} {1}\end{array}\right)$$
$\therefore$

$$\left|\begin{array}{ccc}
1& 1 &1\\
1 & 2 &1\\ 3 & 1 & 1\end{array}\right|^{-1}=\frac{1}{-2}\times
\left(\begin{array}{ccc}
\textcolor{OliveGreen}{1}&\textcolor{Brown} {0} &\textcolor{RedViolet}{-1}\\
\textcolor{Magenta}{2} &\textcolor{Orange} {-2}
&\textcolor{Fuchsia}{0}\\ \textcolor{Violet}{-5} &
\textcolor{Blue}{2} &\textcolor{Emerald}
{1}\end{array}\right)=\left(\begin{array}{ccc}
\textcolor{OliveGreen}{-\frac{1}{2}}&\textcolor{Brown} {0} &\textcolor{RedViolet}{\frac{1}{2}}\\
\textcolor{Magenta}{-1} &\textcolor{Orange} {1}
&\textcolor{Fuchsia}{0}\\\textcolor{Violet}{\frac{5}{2}} &
\textcolor{Blue}{-1} &\textcolor{Emerald}
{-\frac{1}{2}}\end{array}\right)$$ $\therefore$ las parciales son:
$$\frac{\partial u}{\partial x}(q)=-\frac{1}{2}\quad \frac{\partial u}{\partial y}(q)=0\quad \frac{\partial u}{\partial z}(q)=\frac{1}{2}$$
$$\frac{\partial v}{\partial x}(q)=-1\quad \frac{\partial v}{\partial y}(q)=1\quad \frac{\partial v}{\partial z}(q)=0$$
$$\frac{\partial w}{\partial x}(q)=\frac{5}{2}\quad \frac{\partial w}{\partial y}(q)=-1\quad \frac{\partial w}{\partial z}(q)=-\frac{1}{2}$$